Matemáticas, pregunta formulada por wasonxd, hace 4 meses

simplifica el siguiente esquema [~(p→q)→~(q→p)]^(~pvq)​

Respuestas a la pregunta

Contestado por sirleisausen
4

ai está lá respuesta espero tener ajudado

Adjuntos:
Otras preguntas